[問題] 我對有簡併能階的證明哪裡錯了Orz

看板Physics作者 (PE)時間7年前 (2016/10/12 21:34), 7年前編輯推噓1(1013)
留言14則, 2人參與, 最新討論串1/3 (看更多)
【出處】(習題或問題的出處) Sakurai Quantum Mechanics 第一章第17題 【題目】(題目的文字敘述,如有圖片亦可提供圖片) Two observables A1 and A2, which do not involve time explicitly, are known not to commute, [A1,A2] \neq 0 yet we also know that A1 and A2 both commute with the Hamiltonian: [A1,H] = 0, [A2,H] = 0. Prove that the energy eigenstates are, in general, degenerate. Are there exceptions? As an example, you may think of the central-force problem H=p2/2m+V(r), with A1 →Lz,A2 →Lx. 也請看一下附件,內有完整題目與我的詳細作法 https://www.dropbox.com/s/m6ab8zoo0ay6f9d/ask.pdf?dl=0 【瓶頸】(解題瓶頸或思考脈絡,請盡量詳述以利回答者知道要從何處講解指導) ---稍微說明一下我的作法--- 我先假設一個Hamiltonian的特徵值方程式,接著藉由[A1,H]=[A2,H]=0, 來推得另一個特徵值方程式,如此一來,應該就能接著討論它們具有 相同特徵值,但特徵態卻不同的情況,也就是能階具有簡併態的情況。 不過,就在我討論到(5)時..我認為,如果|Φ〉與|ψ〉是不一樣的特徵態, 那麼它們就不能具有倍數關係,也就是(5)的結論。因此,我最後得出, 「如果|ψ〉不是A的特徵態,但又恰好是Hamiltonian的特徵態,那麼 A|ψ〉就會是|ψ〉的簡併態,其中A可代換為A1或A2。」 這規則是正確的嗎?囧... -------------------------- 今天跟同學討論後,發現我的(4)似乎不能推得(5)。我後來仔細想想, 我覺得我是為了讓(4)成立,並且能階具有簡併,所以才不得不得出(5) 的結論。這也是為什麼後面我整理出一個規則的原因。 簡單來說,我還是不明白為什麼我到目前為止的推理是錯的 QQ 不好意思一直麻煩我同學,所以上來發問了,還請各位大大幫忙一下,謝謝Orz -- ※ 發信站: 批踢踢實業坊(ptt.cc), 來自: 140.112.250.76 ※ 文章網址: https://www.ptt.cc/bbs/Physics/M.1476279251.A.7D2.html

10/12 22:16, , 1F
為什麼4不能推到5?
10/12 22:16, 1F
大哥你好,小弟我也不太清楚。原本想法是說,因為我討論的「A」是任何與 Hamiltonian 相容的(compatible)運算子,所以,不是所有的「A」都會滿足 (5)。也就是說,也許|ψ〉不是A1的特徵態,也就滿足(5),但也許|ψ〉 A2的特徵態,這樣就不滿足(5)了。 然而,我後來仔細想想。我承認上述情況是可能的,然而我的目的是要找出 使得|ψ〉具有簡併態的條件,所以我為了排除沒有簡併態的情況,我得出了 |ψ〉必須不是A的特徵態的結論。 不知道這樣推理站得住腳嗎?真的很感謝大大回覆Orz... ※ 編輯: Philethan (140.112.250.76), 10/12/2016 22:23:23

10/13 00:58, , 2F
我覺得是因為沒有把|ψ〉與A扯上關係才遇到這個問題的。
10/13 00:58, 2F

10/13 01:00, , 3F
|ψ〉一開始就假設成A_1和H的共同eigenket(commute op.可
10/13 01:00, 3F

10/13 01:04, , 4F
以有共同的basis)。然後用[A_1,A_2]作用在|ψ〉上,去討論
10/13 01:04, 4F

10/13 01:05, , 5F
A1|ψ〉是否是A2的eigenket,如果是、而且所有的|ψ〉都是
10/13 01:05, 5F

10/13 01:07, , 6F
A2的eigenket,那會導致A1、A2 commute,所以某個|ψ〉和
10/13 01:07, 6F

10/13 01:10, , 7F
A2|ψ〉就線性獨立,但他們有相同的能量。所以能量簡併。
10/13 01:10, 7F

10/13 01:10, , 8F
01:05那一行開頭的A1是多餘的。
10/13 01:10, 8F

10/13 01:12, , 9F
而有時候"能量沒簡併"是可以允許的,因為只要A1,A2在H的
10/13 01:12, 9F

10/13 01:13, , 10F
某個eigenspace上面交換,那個eigenspace就有可能是一維。
10/13 01:13, 10F

10/13 04:35, , 11F
Vulpix說的很清楚了,|ψ〉不可能同時是A1和A2的特徵態
10/13 04:35, 11F

10/13 04:35, , 12F
不然A1和A2就commute了
10/13 04:35, 12F

10/13 04:36, , 13F
所以你的(4)其實得到|ψ〉, A1|ψ〉, A2|ψ〉三個波函數
10/13 04:36, 13F

10/13 04:37, , 14F
其中至少有兩個不平行(只差一個常數)
10/13 04:37, 14F
非常感謝樓上兩位大大的幫忙!小弟這幾天作業比較多,還在趕明天的實驗結報, 所以明後天再好好回兩位大大,真的很感謝Orz... ※ 編輯: Philethan (140.112.25.105), 10/13/2016 20:41:55
文章代碼(AID): #1N_ZlJVI (Physics)
文章代碼(AID): #1N_ZlJVI (Physics)